8

Let $X$ be a Banach space. Suppose that for any sequence of functionals $(\phi_n) \subseteq X^*$ we have that $\phi_n$ converges weakly to some $\phi \in X^*$ if and only if $\phi_n$ converges weakly* to $\phi.$

We know that the weak and weak* topologies on $X^*$ coincide if and only if $X$ is reflexive. Since two topological spaces that have the same convergent sequences do not need to be equal, our $X$ above does not need the weak and weak* topologies to coincide.

So what can we say about such an $X$ above? Is it still reflexive?

gerw
  • 33,373
N. Geis
  • 181
  • The topology of a metrizable space is determined by the convergence of its sequences. – Berci Feb 17 '18 at 00:37
  • 1
    @Berci AFAIK neither weak nor weak* topology are metrizable. – freakish Jun 20 '22 at 13:59
  • 1
    Here is one special case I guess: Suppose $X$ admits a pre-dual $Y$ that is also separable, then your property holds iff $X$ is reflexive. – s.harp Jun 20 '22 at 13:59
  • 2
    Maybe use https://en.wikipedia.org/wiki/Eberlein–Šmulian_theorem ? – GEdgar Jun 20 '22 at 15:42
  • My guess for a counterexample would be $X = C(K)$ where $K$ is compact but not sequentially compact. Examples $K = \beta \mathbb N$ and $K = [0,1]^E$ with $|E| = \mathfrak c$. – GEdgar Jun 25 '22 at 15:48
  • 1
    If even the bounty does not help to answer the question, it might be worthwhile to try on MathOverflow. Bill Johnson probably knows the answer. – Jochen Jun 29 '22 at 15:11
  • 1
    @Jochen: Done, see https://mathoverflow.net/questions/425811. – gerw Jul 01 '22 at 07:18

3 Answers3

1

Here is an argument for the following case:

Suppose $X$ is separable and it admits a predual $X_\#$. Then $X$ is reflexive iff every weak* convergent sequence in $X^*$ is weak convergent.

Begin with the image of the unit ball $X_\#$ in $X^*$, this image is weak closed (since its norm closed + convex) but weak* dense in the unit ball of $X^*$ (by the Goldstine theorem). Weak closed clearly implies weak sequentially closed.

Since $X$ is separable the weak* topology on the unit ball of $X^*$ is metrisable, so every element in the unit ball can be weak* approximated by a sequence in $X_\#$. So unless $X_\#=X^*$ (ie unless $X$ is reflexive) there are sequences that are weak* convergent but not weakly convergent.

s.harp
  • 22,576
  • The weak$^$ topology on the dual of a infinite dimensional Banach space is never* metrizable: It is sequentially complete because of Banach-Steinhaus and if it were metrizable it would be completely metrizable. But then Baire's theorem (or the open mapping theorem) would imply that this weak$^*$ topology coincides with the dual norm topology which is not true. – Jochen Jun 25 '22 at 13:27
  • It is of course true that the dual unit ball with the weak$^*$ topology is metrizable. – Jochen Jun 25 '22 at 13:28
  • @Jochen thank you for the correction. I think the argument itself does not change if one shifts the focus to the unit ball, and edited the answer. Is there something else I overlooked? – s.harp Jun 25 '22 at 15:41
  • If $X$ is separable and in $X^$ weak and weak$^$ convergence of sequences are equivalent then $X$ is reflexive because so is $X^$. As commented by @GEdgar this follows from Eberlein-Smulian (weak compactness = weak sequential compactness) and the metrizability of the dual unit ball in the weak $^$ topology (which is thus sequentially compact). – Jochen Jun 25 '22 at 16:57
  • I think that your arguments are also fine (and more elementary than Eberlein-Smulian). – Jochen Jun 25 '22 at 17:26
1

Here is an expansion of the comment by GEdgar:

Let $X$ be separable and suppose that weak and weak-* convergence of sequences coincides on $X^*$. Let $B \subset X^*$ be the (closed) unit ball.

Since $X$ is separable, the Sequential Banach–Alaoglu theorem implies that $B$ is sequentially weak-* compact. Since weak and weak-* convergent sequences coincide in $X^*$, it follows that $B$ is sequentially weakly compact. Now, the Eberlein–Šmulian theorem implies that $B$ is weakly compact. Finally, Kakutani's theorem yields that $X^*$ (and, consequently, $X$) is reflexive.

Moreover, it is possible to substitute the application of the Eberlein–Šmulian theorem and Kakutani's theorem by James' theorem: Let $f \in X^{**}$ be a linear and continuous functional on $X^*$. Since $B$ is sequentially weakly compact, it is easy to see that the supremum of $f$ is attained on $B$. Thus, James' theorem implies the reflexivity of $X^*$ (and, consequently of $X$).

gerw
  • 33,373
0

This, too, is a partial answer. I originally wrote it to argue that GEdgar's suggestion of $C(\beta\mathbb{N})$ and similar are not counterexamples (i.e., your hypothetical equivalence holds for them), but flubbed a key point. Nevertheless, my argument shows (in rather elementary fashion) that a counterexample should have to be rather exotic.

Specifically, let $K$ be any infinite compact Hausdorff space — say, $\beta\mathbb{N}$, $[0,1]^{\mathbb{R}}$, or just $\omega_1+1$ with the ordinal topology. I will show that if $K$ admits a nontrivial convergent sequence in the sense of this paper — that is, a convergent sequence that is not eventually constant — then $K$ cannot be a counterexample. Not all infinite compact Hausdorff spaces admit such a sequence because they need not be sequentially compact; in particular, such is the case for $\beta\mathbb{N}$.

Anyways, on to the proof!

Pick a maximal set $S$ of mutually singular measures on the Borel $\sigma$-algebra associated with the topology on $K$; $S$ always exists by Zorn's Lemma. Endow $S$ with the discrete topology and place the Borel $\sigma$-algebra on $S\times K$; the latter has a natural measure, given by $$\lambda(A)=\sum_{s\in S}{s(\{x:(s,x)\in A\})}$$

Now, $C(K)^*$ is the space of Borel-regular measures on $K$. By Lebesgue decomposition and Radon-Nikodym, $T:C(K)^*\to L^1(S\times K)$; $$T(\mu)(s,x)=\frac{d\mu}{ds}(x)$$ (where the Radon-Nikodym derivative vanishes if $\mu\perp s$) is an isometric isomorphism. Correspondingly $C(K)^{**}\cong L^{\infty}(S\times K)$, and I now unabashedly identify across both isomorphisms.

In particular, pick any sequence $\{\mu_n\}_{n=0}^{\infty}\in L^1(S\times K)^{\omega_0}$. $\{\mu_n\}_n\overset{*}{\rightharpoonup}0$ iff, for all $f\in C(K)$, we have $$\lim_{n\to\infty}{\iint_{S\times K}{f(x)\mu_n(s,x)\,d^2(s,x)}}=0$$ and $\{\mu_n\}_n\rightharpoonup0$ iff, for all $f\in L^{\infty}(S\times K)$, we have $$\lim_{n\to\infty}{\iint_{S\times K}{f(s,x)\mu_n(s,x)\,d^2(s,x)}}=0$$

With those preliminaries in hand, an easy construction now shows that the topologies of weak and weak-$*$ convergence do not coincide on sequences in $C(K)^*$. Choose any sequence $\{q_n\}_{n=0}^{\infty}\in K^{\omega_0}$ of distinct elements, tending to a limit $q_{\infty}$. W/oLoG, for each $n$, the Dirac mass $\delta_{q_n}\in S$. So let $$\mu_n(s,x)=\begin{cases} 1 & s=\delta_{q_{2n}} \\ -1 & s=\delta_{q_{2n+1}} \\ 0 & \text{otherwise} \end{cases}$$ Then, since any $f\in C(K)$ is continuous at $q_{\infty}$, we can compute $$\iint_{S\times K}{f(x)\mu_n(s,x)\,d^2(s,x)}=f(q_{2n})-f(q_{2n+1})\to0$$ as $n\to\infty$. But just as clearly the pointwise sum $\sum_n{\mu_n}\in L^{\infty}(S\times K)$, so that $$\iint_{S\times K}{\left(\sum_n{\mu_n}\right)(s,x)\mu_n(s,x)\,d^2(s,x)}=2\not\to0$$ as $n\to\infty$.

Jacob Manaker
  • 10,173
  • The comment by M.Gonzales at MO https://mathoverflow.net/q/425811/454 claims that $\ell^\infty$ is a Grothendieck space, i.e. is a counterexample. It seems that is a result of Grothendieck himself, 1953. But, of course,$\ell^\infty = C(\beta\mathbb N)$. – GEdgar Jul 01 '22 at 20:35
  • @GEdgar: Looking through the paper Gonzales linked, some compact Hausdorff spaces don't have enough convergent sequences to apply my construction at the end; at a key point, I replaced "compact" with "sequentially compact" by accident. I've added the missing the hypothesis to my answer. – Jacob Manaker Jul 02 '22 at 20:40
  • @GEdgar: (Also, I hope this answer didn't come across as a personal attack. I wrote this answer because your examples kept on seeming like really good candidates for a counterexample.) – Jacob Manaker Jul 02 '22 at 20:41
  • A $CK)$ space is Grothendieck if and only if it contains no complemented copies of $c_0$. – M.González Jul 03 '22 at 10:19
  • @M.González: Yes, this answer gives one direction of that equality. I think the other direction is given in your link on MO. – Jacob Manaker Jul 05 '22 at 03:32